Login

Welcome, Guest. Please login or register.

April 17, 2024, 12:12:53 am

Author Topic: "Challenge" Math Qs - Can You Figure It Out?  (Read 12508 times)  Share 

0 Members and 1 Guest are viewing this topic.

AlphaZero

  • MOTM: DEC 18
  • Forum Obsessive
  • ***
  • Posts: 352
  • \[\Gamma(z)\Gamma(1-z)=\frac{\pi}{\sin(\pi z)}\]
  • Respect: +160
Re: "Challenge" Math Qs - Can You Figure It Out?
« Reply #30 on: December 26, 2018, 08:15:59 pm »
+2
(Yeah. Told you to be careful :D)
(Background context - basically \( \int \sqrt{\tan x}\,dx\) is a very popular painful integral. It's so popular that people have went on to discovering more efficient ways of solving it, but they lack a lot less intuition than the original approach. But I didn't want to put that one up because that one is either barely easier or 'on par' with the painful one above. So I just decided to revamp up the power a bit instead 8))

I mean, that proof is fair enough at the first year level I think :P it's basically using enumerative combinatorics here instead. To make \(A\) rigourous, you could comment on something along the lines of for a fixed \(k\), let \(A\) be a particular configuration (or sequence) of the \(n\) Ber(p)'s, and then provide an example - e.g. if \(n=2\) and \(k=1\), a particular configuration could be \(X_1 = 0\) and \(X_2 = 1\)). But if you wanna do a bit of reading, look up "sums of independent random variables" and "convolutions" :)

Haha, indeed, even I've tried (and failed) finding more efficient methods for \(\int\sqrt{\tan(x)}\,dx\). (It really is a pain in the a**). I just wanted to see how messy the answer for the fourth root case is :P

For rigorously defining \(A\), I was trying really hard not to give an example lol, though I guess it doesn't hurt to clarify what I mean. I'll be sure to read up on the stuff you mentioned :)
2015\(-\)2017:  VCE
2018\(-\)2021:  Bachelor of Biomedicine and Mathematical Sciences Diploma, University of Melbourne


RuiAce

  • ATAR Notes Lecturer
  • Honorary Moderator
  • Great Wonder of ATAR Notes
  • *******
  • Posts: 8814
  • "All models are wrong, but some are useful."
  • Respect: +2575
Re: "Challenge" Math Qs - Can You Figure It Out?
« Reply #31 on: December 26, 2018, 08:35:54 pm »
+3
Haha, indeed, even I've tried (and failed) finding more efficient methods for \(\int\sqrt{\tan(x)}\,dx\). (It really is a pain in the a**). I just wanted to see how messy the answer for the fourth root case is :P

For rigorously defining \(A\), I was trying really hard not to give an example lol, though I guess it doesn't hurt to clarify what I mean. I'll be sure to read up on the stuff you mentioned :)
Idk maybe I've just done too much combinatorics over my life but I think giving examples are a great thing.to help illustrate your point. Some things are just hard to explain by nature (especially in combinatorics anyway).

For a clever way of approaching \( \int \sqrt{\tan x} dx\) you may want to consider \( \int \sqrt{\tan x} + \sqrt{\cot x}\,dx \) and \( \int \sqrt{\tan x} - \sqrt{\cot x} \,dx\), and taking half the sum of that.

Anyway just gonna bump this because it fell into the previous page
NEXT QUESTION: Ok I'm bored of integrals for now. For the high schoolers:
\[ \text{Find the implied domain of}\\ f(x) = \ln (x^2-4x) \arcsin (x^2-4x-3).\]

Unsplash

  • Forum Regular
  • **
  • Posts: 61
  • Respect: +9
Re: "Challenge" Math Qs - Can You Figure It Out?
« Reply #32 on: December 26, 2018, 11:20:20 pm »
+3
\[ \text{Find the implied domain of}\\ f(x) = \ln (x^2-4x) \arcsin (x^2-4x-3).\]

Hope it's ok if answer this question, even though I just finished high school. Don't plan on studying maths at uni so don't think I have too much of an advantage! However, I'm missing my high school maths subjects already, so I thought I could give it a go...

Also, hopefully its correct!

Spoiler

\[ \text{Let} \: f(x)=g(x) \times h(x) \\ \begin{align*} \therefore g(x) &= ln(x^2-4x) \\ h(x) &= arcsin(x^2-4x-3) \end{align*} \\ \implies \text{Domain of } \: f: \text{Dom} \: g \cap \text{Dom} \: h \]

\[ \text{Inside of a log function must be greater than zero.} \\ \therefore x^2-4x>0 \\ \text{By considering the graph, we obtain:} \\ \text{Dom} \: g: (-\infty,0) \cup (4,\infty) \]

\[ \text{Inside of a arcsin function must be between or equal to, -1 and 1.} \\ \therefore -1 \leq x^2-4x-3 \leq 1 \\ 2 \leq x^2-4x \leq 4 \]

\[ \text{Comparing to above, we identify that this domain is a subset of the domain of} \: g \:  \\ \therefore \text{The domain of} \: f \: \text{is the same as the domain of} \: h.\\ \text{Let the domain of} \: h \: \text{be}\: [a,b] \cup [c,d] \\ \text{Solve for the endpoints:} \]

\[x^2-4x=2 \\ \therefore b=2-\sqrt{6} \quad \& \quad c=2+\sqrt{6}\]

\[x^2-4x=4 \\ \therefore a=2-2\sqrt{2}\quad  \& \quad  d=2+2\sqrt{2}\]

\[\therefore \text{Dom} \: h: [2-2\sqrt{2},2-\sqrt{6}] \cup [2+\sqrt{6},2+2\sqrt{2}]\]

\[\implies  \text{Dom} \: f: [2-2\sqrt{2},2-\sqrt{6}] \cup [2+\sqrt{6},2+2\sqrt{2}]\]


EDITS: Many edits to correct poor LaTeX, still probability mistakes lol.
« Last Edit: December 26, 2018, 11:45:43 pm by FelixHarvey »

lzxnl

  • Victorian
  • ATAR Notes Legend
  • *******
  • Posts: 3432
  • Respect: +215
Re: "Challenge" Math Qs - Can You Figure It Out?
« Reply #33 on: December 27, 2018, 01:30:59 am »
+5
Be careful what you wish for ;)
\[ \int \sqrt[4]{\tan x}\,dx \]
(That's not the next question and I ain't touching it either, but you can if you really want to.)

But yeah I give up on that code. After thinking about it again, if I still need to "check" the obvious before computing the matrix powers (e.g. no 0 in each row and column) it's still gonna be an inefficient algorithm.
______________________________________________________

NEXT QUESTION: Ok I'm bored of integrals for now. For the high schoolers:
\[ \text{Find the implied domain of}\\ f(x) = \ln (x^2-4x) \arcsin (x^2-4x-3).\]
For the university students:
\[ \text{Let }X_1, \dots, X_n\text{ be an i.i.d. sequence of }\operatorname{Ber}(p)\text{ random variables.}\\ \text{Prove that }S := \sum_{i=1}^n X_i \sim \operatorname{Bin}(n,p). \]

This is how I would attempt to integrate tan(x)^(1/4), although my answer looks a little more complicated than Wolfram Alpha's. Amazingly, all the complex parts cancel out exactly. You're seriously making me dig into my bag of tricks now. I'm going to run out soon.

First step: convert to a rational function.

Second step: do partial fractions with a twist. Keep in mind that in this section, outside summation symbols, k is an integer between 0 and 7 inclusive.

Here, if we pick a value of k=p, and make the following substitution, then for all other values of k, the product will contain a term where j=p and this will vanish the product. Hence, the only value of k that doesn't vanish is the term k=p.

Third step: we will evaluate this product using the factor theorem in reverse. The next statement is true because the zeros are exactly the eight eighth roots of 1. Then, the product we want is the product of all 8 terms divided by 1 specific term, which happens to be 0/0, so use L'Hopital.


Fourth step: I don't want complex numbers, so I will group pair together terms where the zeros of the denominator are conjugate pairs. If you plug in k = 0 to k = 7, you'll find that the denominator for k = 0 and k = 7 are conjugates, k = 1 and k  = 6 are conjugates etc. This motivates the next step. Split the sum into two parts, take m = 7 - k and note that instead of summing k from 4 to 7, you're summing m from 0 to 3, just like the other k values.

Fifth step: our partial fraction decomposition is 'complete'; it turns out that the coefficients are real. I really should have done the simplification here, but no worry. Now we have to actually integrate this.

Sixth step: actually make everything real.

I'm pretty proud of myself for coming up with this. Mathematica confirms that my final integration is indeed correct and it looks neat. Kids, this is what maths is about: solving problems you've not ever seen before in class and feeling good about it afterwards. I've never done partial fractions this way before and I've done one question with a similar polynomial trick. I don't think I could tackle many other values of t, however.

Also, define 'binomial distribution' and 'Bernoulli distribution'. I would argue that a binomial distribution is defined as a sum of iid Bernoulli trials, thus your result is true trivially. If you define them in terms of pmfs, generating functions are handy too. This calculation, however, is trivial compared to that integral. Fark. Thank goodness for Mathematica to check my working along the way.
« Last Edit: December 27, 2018, 01:32:34 am by lzxnl »
2012
Mathematical Methods (50) Chinese SL (45~52)

2013
English Language (50) Chemistry (50) Specialist Mathematics (49~54.9) Physics (49) UMEP Physics (96%) ATAR 99.95

2014-2016: University of Melbourne, Bachelor of Science, Diploma in Mathematical Sciences (Applied Maths)

2017-2018: Master of Science (Applied Mathematics)

2019-2024: PhD, MIT (Applied Mathematics)

Accepting students for VCE tutoring in Maths Methods, Specialist Maths and Physics! (and university maths/physics too) PM for more details

RuiAce

  • ATAR Notes Lecturer
  • Honorary Moderator
  • Great Wonder of ATAR Notes
  • *******
  • Posts: 8814
  • "All models are wrong, but some are useful."
  • Respect: +2575
Re: "Challenge" Math Qs - Can You Figure It Out?
« Reply #34 on: December 27, 2018, 09:14:15 am »
+2
Also, define 'binomial distribution' and 'Bernoulli distribution'. I would argue that a binomial distribution is defined as a sum of iid Bernoulli trials, thus your result is true trivially. If you define them in terms of pmfs, generating functions are handy too. This calculation, however, is trivial compared to that integral. Fark. Thank goodness for Mathematica to check my working along the way.
Yep PMFs. MGFs would make the computations doable in just a few lines so that would work. Didn't think about it at the time but I wasn't trying to make that one hard.
Hope it's ok if answer this question, even though I just finished high school. Don't plan on studying maths at uni so don't think I have too much of an advantage! However, I'm missing my high school maths subjects already, so I thought I could give it a go...

Also, hopefully its correct!

Spoiler

\[ \text{Let} \: f(x)=g(x) \times h(x) \\ \begin{align*} \therefore g(x) &= ln(x^2-4x) \\ h(x) &= arcsin(x^2-4x-3) \end{align*} \\ \implies \text{Domain of } \: f: \text{Dom} \: g \cap \text{Dom} \: h \]

\[ \text{Inside of a log function must be greater than zero.} \\ \therefore x^2-4x>0 \\ \text{By considering the graph, we obtain:} \\ \text{Dom} \: g: (-\infty,0) \cup (4,\infty) \]

\[ \text{Inside of a arcsin function must be between or equal to, -1 and 1.} \\ \therefore -1 \leq x^2-4x-3 \leq 1 \\ 2 \leq x^2-4x \leq 4 \]

\[ \text{Comparing to above, we identify that this domain is a subset of the domain of} \: g \:  \\ \therefore \text{The domain of} \: f \: \text{is the same as the domain of} \: h.\\ \text{Let the domain of} \: h \: \text{be}\: [a,b] \cup [c,d] \\ \text{Solve for the endpoints:} \]

\[x^2-4x=2 \\ \therefore b=2-\sqrt{6} \quad \& \quad c=2+\sqrt{6}\]

\[x^2-4x=4 \\ \therefore a=2-2\sqrt{2}\quad  \& \quad  d=2+2\sqrt{2}\]

\[\therefore \text{Dom} \: h: [2-2\sqrt{2},2-\sqrt{6}] \cup [2+\sqrt{6},2+2\sqrt{2}]\]

\[\implies  \text{Dom} \: f: [2-2\sqrt{2},2-\sqrt{6}] \cup [2+\sqrt{6},2+2\sqrt{2}]\]


EDITS: Many edits to correct poor LaTeX, still probability mistakes lol.
It looked right after you fixed it :D

Next question: Prove the Cauchy-Schwarz inequality for vectors in \(\mathbb{R}^3\) (however you like, but if there's a VCE method then go with that)
\[ |\underset{\sim}{a} \cdot \underset{\sim}{b}| \leq | \underset{\sim}{a} | |\underset{\sim}{b}| \]
(Someone else take over writing questions after this one pls.)

Edit: Yeah no square oops
« Last Edit: December 27, 2018, 03:25:09 pm by RuiAce »

lzxnl

  • Victorian
  • ATAR Notes Legend
  • *******
  • Posts: 3432
  • Respect: +215
Re: "Challenge" Math Qs - Can You Figure It Out?
« Reply #35 on: December 27, 2018, 01:30:49 pm »
+4
Yep PMFs. MGFs would make the computations doable in just a few lines so that would work. Didn't think about it at the time but I wasn't trying to make that one hard.It looked right after you fixed it :D

Next question: Prove the Cauchy-Schwarz inequality for vectors in \(\mathbb{R}^3\) (however you like, but if there's a VCE method then go with that)
\[ |\underset{\sim}{a} \cdot \underset{\sim}{b}|^2 \leq | \underset{\sim}{a} | |\underset{\sim}{b}| \]
(Someone else take over writing questions after this one pls.)
I'm going to leave this one for someone else. The simplest method for doing this would be related to thinking about why this is true geometrically (I don't think RuiAce wants someone to say 'because \(\cos(\theta) \in [-1,1]\)' btw; this should motivate a proof, however, which is certainly doable at VCE level).

Here's a fun question for you guys. Can \( \int_0^\infty f(x)\,dx\) exist if \(f(x)\) does not decay to zero as \(x\rightarrow\infty\)?
2012
Mathematical Methods (50) Chinese SL (45~52)

2013
English Language (50) Chemistry (50) Specialist Mathematics (49~54.9) Physics (49) UMEP Physics (96%) ATAR 99.95

2014-2016: University of Melbourne, Bachelor of Science, Diploma in Mathematical Sciences (Applied Maths)

2017-2018: Master of Science (Applied Mathematics)

2019-2024: PhD, MIT (Applied Mathematics)

Accepting students for VCE tutoring in Maths Methods, Specialist Maths and Physics! (and university maths/physics too) PM for more details

AlphaZero

  • MOTM: DEC 18
  • Forum Obsessive
  • ***
  • Posts: 352
  • \[\Gamma(z)\Gamma(1-z)=\frac{\pi}{\sin(\pi z)}\]
  • Respect: +160
Re: "Challenge" Math Qs - Can You Figure It Out?
« Reply #36 on: December 27, 2018, 03:22:33 pm »
+2
\[ |\underset{\sim}{a} \cdot \underset{\sim}{b}|^2 \leq | \underset{\sim}{a} | |\underset{\sim}{b}| \]
Shouldn't the RHS be \(\left|\underset{\sim}{\text{a}}\right|^2\left|\underset{\sim}{\text{b}}\right|^2\)?

Very cheap solution (sorry)
It's clear that the inequality holds if \(\underset{\sim}{\text{a}}=\underset{\sim}{0}\) or \(\underset{\sim}{\text{b}}=\underset{\sim}{0}\), so we will consider \(\underset{\sim}{\text{a}}\neq \underset{\sim}{0}\) and \(\underset{\sim}{\text{b}}\neq \underset{\sim}{0}\).
\begin{align*}\left|\underset{\sim}{\text{a}}\cdot\underset{\sim}{\text{b}}\right|&=\left|\left|\underset{\sim}{\text{a}}\right|\left|\underset{\sim}{\text{b}}\right|\cos(\theta)\right|\\
&=\left|\underset{\sim}{\text{a}}\right|\left|\underset{\sim}{\text{b}}\right|\left|\cos(\theta)\right|\\
&\leq \left|\underset{\sim}{\text{a}}\right|\left|\underset{\sim}{\text{b}}\right|\quad \Big(0\leq\left|\cos(\theta)\right|\leq 1\Big)\end{align*}


Here's a fun question for you guys. Can \( \int_0^\infty f(x)\,dx\) exist if \(f(x)\) does not decay to zero as \(x\rightarrow\infty\)?

Fun example I came across some time ago
\[\text{Let }f(x)=\begin{cases}1, & x\in[n,\ n+2^{-n}]\\ 0, & \text{elsewhere}\end{cases},\ \ n\in\mathbb{N}\cup\{0\}.\] \[\int_0^\infty f(x)\;dx=\sum_{n=0}^\infty 2^{-n}=2\ \text{ and }\ f(x)\nrightarrow 0\ \text{as}\ x\to\infty.\] So, the answer is yes, it is possible.


Next Questions

For highschool students:
Find the magnitude of the acute angle formed by the two lines in \(\mathbb{R}^2\) given by
\[5\sqrt{3}x+9y=9\ \ \text{and}\ -\!\sqrt{3}x+6y=6.\]
For uni students:
Prove Lagrange's identity
\[\|\mathbf{u}\times\mathbf{v}\|^2=\|\mathbf{u}\|^2\|\mathbf{v}\|^2-(\mathbf{u}\cdot\mathbf{v})^2.\]
Or, if you're not a fan of the algebra required in the above question:

Solve for \(y(t)\) the initial value problem \[\frac{dy}{dt}=\sin(y),\quad y(0)=\alpha\in\mathbb{R}\setminus\{k\pi\mid k\in\mathbb{Z}\}.\] Then, show that \[\lim_{t\to\infty}y(t)=(2m+1)\pi,\quad \text{for some }m\in\mathbb{Z}.\]
« Last Edit: December 27, 2018, 03:25:57 pm by dantraicos »
2015\(-\)2017:  VCE
2018\(-\)2021:  Bachelor of Biomedicine and Mathematical Sciences Diploma, University of Melbourne


RuiAce

  • ATAR Notes Lecturer
  • Honorary Moderator
  • Great Wonder of ATAR Notes
  • *******
  • Posts: 8814
  • "All models are wrong, but some are useful."
  • Respect: +2575
Re: "Challenge" Math Qs - Can You Figure It Out?
« Reply #37 on: December 27, 2018, 03:42:53 pm »
+2
Shouldn't the RHS be \(\left|\underset{\sim}{\text{a}}\right|^2\left|\underset{\sim}{\text{b}}\right|^2\)?

Very cheap solution (sorry)
It's clear that the inequality holds if \(\underset{\sim}{\text{a}}=\underset{\sim}{0}\) or \(\underset{\sim}{\text{b}}=\underset{\sim}{0}\), so we will consider \(\underset{\sim}{\text{a}}\neq \underset{\sim}{0}\) and \(\underset{\sim}{\text{b}}\neq \underset{\sim}{0}\).
\begin{align*}\left|\underset{\sim}{\text{a}}\cdot\underset{\sim}{\text{b}}\right|&=\left|\left|\underset{\sim}{\text{a}}\right|\left|\underset{\sim}{\text{b}}\right|\cos(\theta)\right|\\
&=\left|\underset{\sim}{\text{a}}\right|\left|\underset{\sim}{\text{b}}\right|\left|\cos(\theta)\right|\\
&\leq \left|\underset{\sim}{\text{a}}\right|\left|\underset{\sim}{\text{b}}\right|\quad \Big(0\leq\left|\cos(\theta)\right|\leq 1\Big)\end{align*}


Fun example I came across some time ago
\[\text{Let }f(x)=\begin{cases}1, & x\in[n,\ n+2^{-n}]\\ 0, & \text{elsewhere}\end{cases},\ \ n\in\mathbb{N}\cup\{0\}.\] \[\int_0^\infty f(x)\;dx=\sum_{n=0}^\infty 2^{-n}=2\ \text{ and }\ f(x)\nrightarrow 0\ \text{as}\ x\to\infty.\] So, the answer is yes, it is possible.


Next Questions

For highschool students:
Find the magnitude of the acute angle formed by the two lines in \(\mathbb{R}^2\) given by
\[5\sqrt{3}x+9y=9\ \ \text{and}\ -\!\sqrt{3}x+6y=6.\]
For uni students:
Prove Lagrange's identity
\[\|\mathbf{u}\times\mathbf{v}\|^2=\|\mathbf{u}\|^2\|\mathbf{v}\|^2-(\mathbf{u}\cdot\mathbf{v})^2.\]
Or, if you're not a fan of the algebra required in the above question:

Solve for \(y(t)\) the initial value problem \[\frac{dy}{dt}=\sin(y),\quad y(0)=\alpha\in\mathbb{R}\setminus\{k\pi\mid k\in\mathbb{Z}\}.\] Then, show that \[\lim_{t\to\infty}y(t)=(2m+1)\pi,\quad \text{for some }m\in\mathbb{Z}.\]
Maybe I should've put Minkowski's instead.

But yeah, those rectangles remind me about the fun times in real analysis :P exploiting the natural numbers for the triangles/rectangles. (I think I usually used triangles to preserve the continuity of the function, but it wasn't specified here so no matter.) There's also the fancy complex analysis answer of \( \int_0^\infty \cos (x^2) \,dx \) but of course that takes more effort to derive an answer to.
\begin{align*}&\quad\| \mathbf{u}\|^2 \| \mathbf{v} \|^2 - (\mathbf{u} \cdot \mathbf{v})^2\\ &= (u_1^2+u_2^2+u_3^2)(v_1^2+v_2^2+v_3^2) - (u_1v_1+u_2v_2+u_3v_3)^2\\ &= (u_1^2v_1^2+u_2^2v_2^2+u_3^2v_3^2) + (u_1^2v_2^2 + u_1^2v_3^2 + u_2^2v_1^2 + u_2^2 v_3^2 + u_3^2 v_1^2 + u_3^2 v_2^2)\\ &\qquad - (u_1^2 v_1^2 + u_2^2 v_2^2 + u_3^2 v_3^2) - 2(u_1v_1u_2v_2 + u_1v_1u_3v_3 + u_2v_2u_3v_3)\\ &= (u_2^2v_3^2 - 2u_2v_3u_3v_2 + u_3^2v_2^2) + (u_3^2 v_1^2 - 2u_3v_1u_1v_3 + u_1^2v_3^2) + (u_1^2v_2^2 - 2u_1v_2u_2v_1 + u_2^2v_1^2)\\ &= (u_2v_3-u_3v_2)^2 + (u_3v_1-u_1v_3)^2 + (u_1v_2-u_2v_1)^2\\ &= \| \mathbf{u} \times \mathbf{v} \|^2\end{align*}

lzxnl

  • Victorian
  • ATAR Notes Legend
  • *******
  • Posts: 3432
  • Respect: +215
Re: "Challenge" Math Qs - Can You Figure It Out?
« Reply #38 on: December 27, 2018, 04:09:20 pm »
+1
Shouldn't the RHS be \(\left|\underset{\sim}{\text{a}}\right|^2\left|\underset{\sim}{\text{b}}\right|^2\)?

Very cheap solution (sorry)
It's clear that the inequality holds if \(\underset{\sim}{\text{a}}=\underset{\sim}{0}\) or \(\underset{\sim}{\text{b}}=\underset{\sim}{0}\), so we will consider \(\underset{\sim}{\text{a}}\neq \underset{\sim}{0}\) and \(\underset{\sim}{\text{b}}\neq \underset{\sim}{0}\).
\begin{align*}\left|\underset{\sim}{\text{a}}\cdot\underset{\sim}{\text{b}}\right|&=\left|\left|\underset{\sim}{\text{a}}\right|\left|\underset{\sim}{\text{b}}\right|\cos(\theta)\right|\\
&=\left|\underset{\sim}{\text{a}}\right|\left|\underset{\sim}{\text{b}}\right|\left|\cos(\theta)\right|\\
&\leq \left|\underset{\sim}{\text{a}}\right|\left|\underset{\sim}{\text{b}}\right|\quad \Big(0\leq\left|\cos(\theta)\right|\leq 1\Big)\end{align*}


Fun example I came across some time ago
\[\text{Let }f(x)=\begin{cases}1, & x\in[n,\ n+2^{-n}]\\ 0, & \text{elsewhere}\end{cases},\ \ n\in\mathbb{N}\cup\{0\}.\] \[\int_0^\infty f(x)\;dx=\sum_{n=0}^\infty 2^{-n}=2\ \text{ and }\ f(x)\nrightarrow 0\ \text{as}\ x\to\infty.\] So, the answer is yes, it is possible.


Next Questions

For highschool students:
Find the magnitude of the acute angle formed by the two lines in \(\mathbb{R}^2\) given by
\[5\sqrt{3}x+9y=9\ \ \text{and}\ -\!\sqrt{3}x+6y=6.\]
For uni students:
Prove Lagrange's identity
\[\|\mathbf{u}\times\mathbf{v}\|^2=\|\mathbf{u}\|^2\|\mathbf{v}\|^2-(\mathbf{u}\cdot\mathbf{v})^2.\]
Or, if you're not a fan of the algebra required in the above question:

Solve for \(y(t)\) the initial value problem \[\frac{dy}{dt}=\sin(y),\quad y(0)=\alpha\in\mathbb{R}\setminus\{k\pi\mid k\in\mathbb{Z}\}.\] Then, show that \[\lim_{t\to\infty}y(t)=(2m+1)\pi,\quad \text{for some }m\in\mathbb{Z}.\]

I am not a fan of that solution :P although I should really have specified conditions on f, like differentiability. How about proving that RuiAce's solution, \(\int_0^\infty \cos(x^2)\,dx\) actually converges? That's a fairly simple exercise.

RuiAce, you should have specified a general inner product tbh.

Lagrange's identity is really just \(\sin^2(x) + \cos^2(x) = 1\).

The asymptotics of your IVP problem is just a matter of showing that there are stable fixed points. I'll leave that to someone else and do the integration.

etc
2012
Mathematical Methods (50) Chinese SL (45~52)

2013
English Language (50) Chemistry (50) Specialist Mathematics (49~54.9) Physics (49) UMEP Physics (96%) ATAR 99.95

2014-2016: University of Melbourne, Bachelor of Science, Diploma in Mathematical Sciences (Applied Maths)

2017-2018: Master of Science (Applied Mathematics)

2019-2024: PhD, MIT (Applied Mathematics)

Accepting students for VCE tutoring in Maths Methods, Specialist Maths and Physics! (and university maths/physics too) PM for more details

fun_jirachi

  • MOTM: AUG 18
  • HSC Moderator
  • Part of the furniture
  • *****
  • Posts: 1068
  • All doom and Gloom.
  • Respect: +710
Re: "Challenge" Math Qs - Can You Figure It Out?
« Reply #39 on: December 27, 2018, 06:49:09 pm »
+2
For highschool students:
Find the magnitude of the acute angle formed by the two lines in \(\mathbb{R}^2\) given by
\[5\sqrt{3}x+9y=9\ \ \text{and}\ -\!\sqrt{3}x+6y=6.\]



Was thinking I might get baited because the answer seemed a bit too clean, but here is what I got anyway. :)
« Last Edit: December 27, 2018, 06:58:24 pm by fun_jirachi »
Spoiler
HSC 2018: Mod Hist [88] | 2U Maths [98]
HSC 2019: Physics [92] | Chemistry [93] | English Adv [87] | 3U Maths [98] | 4U Maths [97]
ATAR: 99.05

UCAT: 3310 - VR [740] | DM [890] | QR [880] | AR [800]
Guide Links:
Subject Acceleration (2018)
UCAT Question Compilation/FAQ (2020)
Asking good questions

RuiAce

  • ATAR Notes Lecturer
  • Honorary Moderator
  • Great Wonder of ATAR Notes
  • *******
  • Posts: 8814
  • "All models are wrong, but some are useful."
  • Respect: +2575
Re: "Challenge" Math Qs - Can You Figure It Out?
« Reply #40 on: December 28, 2018, 09:09:44 am »
+3
Yeah I wanted to keep the question within the bounds of VCE terminology so I tried just doing the R^3 case, but now I see that was a mistake lol.

Just gonna prove the convergence because ceebs looking up how to solve that integral right now.
\[ x = \sqrt{u} \implies dx = \frac1{2\sqrt{u}}\,du.\\ u = \frac{1}{s} \implies ds = -\frac{1}{s^2}\,ds.\\ \begin{align*} \int_0^\infty \cos (x^2)\,dx &= \int_0^\infty\frac{1}{2}u^{-1/2}\cos (u)\,du\\ &= \int_0^1 \frac12 u^{-1/2} \cos (u^2)\,du + \int_1^\infty \frac12 u^{-1/2}\cos u\,du \\ &= \int_1^\infty \frac12 s^{1/2} \cos \left( \frac{1}{s^2} \right) s^{-2}\,ds +\int_1^\infty \frac12 u^{-1/2}\cos u\,du \end{align*} \]
\[ \text{For the second integral}\\ \begin{align*} \int_1^\infty u^{-1/2} \cos u\,du &= u^{-1/2} \sin u \big|_1^\infty + \frac12 \int_1^\infty u^{-3/2} \sin u\,du. \end{align*}\\ \text{The former is finite and equals }\sin 1\\ \text{and the latter converges via comparison with the }p\text{-integral }\int_1^\infty u^{-3/2}\,du. \]
Squeeze theorem limit used
\[ 0 = \lim_{u\to \infty} -u^{-1/2} \leq \lim_{u\to \infty} u^{-1/2}\sin u \leq \lim_{u\to \infty} u^{-1/2} = 0\\ \implies \lim_{u\to \infty} u^{-1/2}\sin u = 0 \]
The first integral converges using the same comparison. So puzzling everything together the whole thing converges.


Was thinking I might get baited because the answer seemed a bit too clean, but here is what I got anyway. :)
It's harder for the VCE students because they don't have that nice formula that we do. In the HSC your approach would be the intended approach
_________________________________________________________

Continuing,
\begin{align*}e^t &= \frac{\tan \frac{y}{2}}{\tan \frac{\alpha}{2}} \\ y &= 2 \arctan \left( e^t \tan \frac{\alpha}{2} \right) + 2m\pi \end{align*}
where \(m \in \mathbb{Z}\).
\[ \text{As }\alpha\text{ is not an integer multiple of }\pi,\\ \tan \frac{\alpha}{2} \text{ is well defined and not equal to 0.}\\ \text{If }\tan \alpha > 0,\, 2\arctan \left( e^t \tan \frac{\alpha}{2} \right) \to 2\times \frac\pi2 = \pi\text{ and we're done.}\\ \text{Otherwise it just approaches }-\frac\pi2\text{ instead, but then sub }m=n+1\text{ and we're also done.} \]
_________________________________________________________

Next question:
High school:
\[ \text{1. Prove that }\sum_{k=1}^n k^2 = \frac16 n (n+1)(2n+1)\\ \text{2. Now derive it from scratch.} \]
First year/Second year university:
\[ \text{Show that in general, }\lim_{x\to a} \lim_{y\to b} f(x,y) \neq \lim_{y\to b} \lim_{x\to a} f(x,y)\\ \text{where }a, b \in \mathbb{R} \cup \{\infty\} \]
Try to construct a routine counterexample, and also a creative counterexample.

Beyond:
Label the vertices of \(K_5\) 1, 2, 3, 4 and 5, where \(K_n\) is the complete graph on \(n\) vertices. Suppose that two Euler circuits are the same if their sequences of edges are up to the same rotational symmetry. (For example 1-2-3-4-5-1-3-5-2-4-1 would be the same as 2-3-4-5-1-3-5-2-4-1-2).
1. Explain why \(K_3\) has only two distinct Euler circuits (easy warm-up)
2. Carefully prove that \(K_5\) has 264 distinct Euler circuits.
« Last Edit: December 28, 2018, 10:14:15 am by RuiAce »

lzxnl

  • Victorian
  • ATAR Notes Legend
  • *******
  • Posts: 3432
  • Respect: +215
Re: "Challenge" Math Qs - Can You Figure It Out?
« Reply #41 on: December 28, 2018, 11:47:10 am »
+1
Yeah I wanted to keep the question within the bounds of VCE terminology so I tried just doing the R^3 case, but now I see that was a mistake lol.

Just gonna prove the convergence because ceebs looking up how to solve that integral right now.
\[ x = \sqrt{u} \implies dx = \frac1{2\sqrt{u}}\,du.\\ u = \frac{1}{s} \implies ds = -\frac{1}{s^2}\,ds.\\ \begin{align*} \int_0^\infty \cos (x^2)\,dx &= \int_0^\infty\frac{1}{2}u^{-1/2}\cos (u)\,du\\ &= \int_0^1 \frac12 u^{-1/2} \cos (u^2)\,du + \int_1^\infty \frac12 u^{-1/2}\cos u\,du \\ &= \int_1^\infty \frac12 s^{1/2} \cos \left( \frac{1}{s^2} \right) s^{-2}\,ds +\int_1^\infty \frac12 u^{-1/2}\cos u\,du \end{align*} \]
\[ \text{For the second integral}\\ \begin{align*} \int_1^\infty u^{-1/2} \cos u\,du &= u^{-1/2} \sin u \big|_1^\infty + \frac12 \int_1^\infty u^{-3/2} \sin u\,du. \end{align*}\\ \text{The former is finite and equals }\sin 1\\ \text{and the latter converges via comparison with the }p\text{-integral }\int_1^\infty u^{-3/2}\,du. \]
Squeeze theorem limit used
\[ 0 = \lim_{u\to \infty} -u^{-1/2} \leq \lim_{u\to \infty} u^{-1/2}\sin u \leq \lim_{u\to \infty} u^{-1/2} = 0\\ \implies \lim_{u\to \infty} u^{-1/2}\sin u = 0 \]
The first integral converges using the same comparison. So puzzling everything together the whole thing converges.It's harder for the VCE students because they don't have that nice formula that we do. In the HSC your approach would be the intended approach
_________________________________________________________

Continuing,
\begin{align*}e^t &= \frac{\tan \frac{y}{2}}{\tan \frac{\alpha}{2}} \\ y &= 2 \arctan \left( e^t \tan \frac{\alpha}{2} \right) + 2m\pi \end{align*}
where \(m \in \mathbb{Z}\).
\[ \text{As }\alpha\text{ is not an integer multiple of }\pi,\\ \tan \frac{\alpha}{2} \text{ is well defined and not equal to 0.}\\ \text{If }\tan \alpha > 0,\, 2\arctan \left( e^t \tan \frac{\alpha}{2} \right) \to 2\times \frac\pi2 = \pi\text{ and we're done.}\\ \text{Otherwise it just approaches }-\frac\pi2\text{ instead, but then sub }m=n+1\text{ and we're also done.} \]
_________________________________________________________

Next question:
High school:
\[ \text{1. Prove that }\sum_{k=1}^n k^2 = \frac16 n (n+1)(2n+1)\\ \text{2. Now derive it from scratch.} \]
First year/Second year university:
\[ \text{Show that in general, }\lim_{x\to a} \lim_{y\to b} f(x,y) \neq \lim_{y\to b} \lim_{x\to a} f(x,y)\\ \text{where }a, b \in \mathbb{R} \cup \{\infty\} \]
Try to construct a routine counterexample, and also a creative counterexample.

Beyond:
Label the vertices of \(K_5\) 1, 2, 3, 4 and 5, where \(K_n\) is the complete graph on \(n\) vertices. Suppose that two Euler circuits are the same if their sequences of edges are up to the same rotational symmetry. (For example 1-2-3-4-5-1-3-5-2-4-1 would be the same as 2-3-4-5-1-3-5-2-4-1-2).
1. Explain why \(K_3\) has only two distinct Euler circuits (easy warm-up)
2. Carefully prove that \(K_5\) has 264 distinct Euler circuits.
Yeah that's how you would prove the convergence. As for actually calculating the integral, you would consider the real part of \(\int_0^\infty e^{ix^2}\,dx\) and consider a sector of angle \(\frac{\pi}{4}\) in the complex plane; the arc at infinity vanishes, the total integral is zero by Cauchy's integral theorem, and the integral along the ray is a Gaussian integral.

I love the sum question. I think I posted a solution to this on a VCE maths thread somewhere.

For the limit question, the cheapest example I can think of is non-uniform convergence, if we take one of x, y to be an index for a sequence of functions.

My graph theory is horrendous so I'm going to sit out the second one. The first one looks like mirror images though. I'm going to refrain from actually solving questions now to give others a shot too.

Here are some for high school students.
1. Prove the cosine rule in trigonometry using properties of the vector dot product
2. Fully expand \(\sin(5x)\) in terms of powers of \(\sin(x)\). Use this result to find \(\sin\left(\frac{2\pi}{5}\right)\). You may encounter a quintic equation. One of the solutions, which you should be able to show isn't \(\sin\left(\frac{2\pi}{y}\right)\), should be easy to find/guess. Factorise that, and the resulting quartic should be easily solvable.
2012
Mathematical Methods (50) Chinese SL (45~52)

2013
English Language (50) Chemistry (50) Specialist Mathematics (49~54.9) Physics (49) UMEP Physics (96%) ATAR 99.95

2014-2016: University of Melbourne, Bachelor of Science, Diploma in Mathematical Sciences (Applied Maths)

2017-2018: Master of Science (Applied Mathematics)

2019-2024: PhD, MIT (Applied Mathematics)

Accepting students for VCE tutoring in Maths Methods, Specialist Maths and Physics! (and university maths/physics too) PM for more details

fun_jirachi

  • MOTM: AUG 18
  • HSC Moderator
  • Part of the furniture
  • *****
  • Posts: 1068
  • All doom and Gloom.
  • Respect: +710
Re: "Challenge" Math Qs - Can You Figure It Out?
« Reply #42 on: December 28, 2018, 02:21:15 pm »
+2
High school:
\[ \text{1. Prove that }\sum_{k=1}^n k^2 = \frac16 n (n+1)(2n+1)\\ \text{2. Now derive it from scratch.} \]

1. Rephrasing the question,


Before I start Q2, am I allowed to know the result from Q1 and work towards it? Or do I have to derive derive it? :(
Spoiler
HSC 2018: Mod Hist [88] | 2U Maths [98]
HSC 2019: Physics [92] | Chemistry [93] | English Adv [87] | 3U Maths [98] | 4U Maths [97]
ATAR: 99.05

UCAT: 3310 - VR [740] | DM [890] | QR [880] | AR [800]
Guide Links:
Subject Acceleration (2018)
UCAT Question Compilation/FAQ (2020)
Asking good questions

RuiAce

  • ATAR Notes Lecturer
  • Honorary Moderator
  • Great Wonder of ATAR Notes
  • *******
  • Posts: 8814
  • "All models are wrong, but some are useful."
  • Respect: +2575
Re: "Challenge" Math Qs - Can You Figure It Out?
« Reply #43 on: December 28, 2018, 02:29:23 pm »
+4
Before I start Q2, am I allowed to know the result from Q1 and work towards it? Or do I have to derive derive it? :(
It says from scratch. So derive derive it. :P

Never anything wrong with rephrasing it but may be worth noting that it can still be done in the sigma notation form.
\[ \text{Assuming that }\sum_{k=1}^K k^2 = \frac{1}{6}K(K+1)(2K+1)\text{ we have}\\ \begin{align*}\sum_{k=1}^{K+1}k^2 &= \sum_{k=1}^K k^2 + (K+1)^2\\ &= \frac16K(K+1)(2K+1) \tag{assumption}\end{align*}\\ \text{and then continue as you've already done so} \]
Basically this is the technique of "pulling terms out of the sum".
« Last Edit: December 28, 2018, 02:38:07 pm by RuiAce »

fun_jirachi

  • MOTM: AUG 18
  • HSC Moderator
  • Part of the furniture
  • *****
  • Posts: 1068
  • All doom and Gloom.
  • Respect: +710
Re: "Challenge" Math Qs - Can You Figure It Out?
« Reply #44 on: December 28, 2018, 03:37:47 pm »
+1
2. Fully expand \(\sin(5x)\) in terms of powers of \(\sin(x)\). Use this result to find \(\sin\left(\frac{2\pi}{5}\right)\). You may encounter a quintic equation. One of the solutions, which you should be able to show isn't \(\sin\left(\frac{2\pi}{y}\right)\), should be easy to find/guess. Factorise that, and the resulting quartic should be easily solvable.



The derivation derivation is coming, this question just seemed a lot friendlier. :)

EDIT: working on that right now, but also if you guys have time can you try and find the mistakes in the code? I've been trying for about 45 minutes (15 minutes typing, 45 minutes looking for errors) and I can't find them, super frustrating :(
« Last Edit: December 28, 2018, 03:47:44 pm by fun_jirachi »
Spoiler
HSC 2018: Mod Hist [88] | 2U Maths [98]
HSC 2019: Physics [92] | Chemistry [93] | English Adv [87] | 3U Maths [98] | 4U Maths [97]
ATAR: 99.05

UCAT: 3310 - VR [740] | DM [890] | QR [880] | AR [800]
Guide Links:
Subject Acceleration (2018)
UCAT Question Compilation/FAQ (2020)
Asking good questions